if Angie’s gross pay for 21.5 hours was $282.08, what was her pay per hour?

Answers

Answer 1

Answer:

$13.12 per hour

Step-by-step explanation:

Take the total pay and divide by the number of hours

$282.08/21.5 hours

$13.12 per hour

Answer 2

Answer:

Step-by-step explanation:

21.5 hours - $282.08

1 - ?

$282.08/21.5 = $13.12


Related Questions

1. In a right triangle, the lengths of the legs are a and b. Find the length of a hypotenuse, if: a=1, b=1; 2. In a right triangle, the length of a hypotenuse is c and the length of one leg is a. Find the length of the other leg, if: c=5, a=3;

Answers

Answer:

1. [tex]c = \sqrt{2}[/tex].

2. b = 4.

Step-by-step explanation:

To solve these two questions, keep the Pythagorean Theorem in mind: [tex]a^2 + b^2 = c^2[/tex]. Also remember that measurements cannot be negative, so we will disregard the negative answers.

1. a = 1, and b = 1. c = ?

[tex]1^2 + 1^2 = c^2[/tex]

[tex]1 + 1 = c^2\\[/tex]

[tex]c^2 = 1 + 1\\c^2 = 2\\\sqrt{c^2} = \sqrt{2}\\c = \sqrt{2}[/tex]

2. a = 3, c = 5. b = ?

[tex]3^2 + b^2 = 5^2\\9 + b^2 = 25\\b^2 = 16\\\sqrt{b^2} = \sqrt{16}\\b = 4[/tex]

Hope this helps!

1)Sheyna drive to the lake and back. It took two hours less time to get there than it did to get back. The average speed on the trip there was 60 mph. The average speed on the way back was 36 mph. How many hours did the trip there take?

Answers

Answer:

8 hours

Step-by-step explanation:

Given:

Sheyna drives to the lake with average speed of 60 mph and

[tex]v_1 = 60\ mph[/tex]

Sheyna drives back from the lake with average speed of 36 mph

[tex]v_2 = 36\ mph[/tex]

It took 2 hours less time to get there than it did to get back.

Let [tex]t_1[/tex] be the time taken to drive to lake.

Let [tex]t_2[/tex] be the time taken to drive back from lake.

[tex]t_2-t_1 = 2[/tex] hrs ..... (1)

To find:

Total time taken = ?

[tex]t_1+t_2 = ?[/tex]

Solution:

Let D be the distance to lake.

Formula for time is given as:

[tex]Time =\dfrac{Distance}{Speed }[/tex]

[tex]t_1 = \dfrac{D}{60}\ hrs[/tex]

[tex]t_2 = \dfrac{D}{36}\ hrs[/tex]

Putting in equation (1):

[tex]\dfrac{D}{36}-\dfrac{D}{60} = 2\\\Rightarrow \dfrac{5D-3D}{180} = 2\\\Rightarrow \dfrac{2D}{180} = 2\\\Rightarrow D = 180\ miles[/tex]

So,

[tex]t_1 = \dfrac{180}{60}\ hrs = 3 \ hrs[/tex]

[tex]t_2 = \dfrac{180}{36}\ hrs = 5\ hrs[/tex]

So, the answer is:

[tex]t_1+t_2 = \bold{8\ hrs}[/tex]

|3x–1|=8 please help!!!!!

Answers

Answer: -3

Add 1 to both sides

[tex]3x-1+1=8+1[/tex]

[tex]3x=9[/tex]

Divide both sides by 3

[tex]3x/3=9/3\\x=3[/tex]

I NEED YOUR HELP PLS

Answers

Answer:

For question 1 you can try dividing each of the value

For instance, you can divide 9 by 25 and see if you get a nice number

e.g. 1/8=0.125, numbers like these

For the second question, you can find the fraction by dividing 1000 starting with the decimal points

e.g 0.650, you would be plotting 650/1000 and you would simplify the fraction to the lowest value any value above the decimal point you can multiply by the denominator and add the nominator value to get your final answer.

Step-by-step explanation:

Answer:

Write the denominator in its prime factors. If the prime factorization of the denominator of a fraction has only factors of 2 and factors of 5, the decimal expression terminates.  If there is any prime factor in the denominator other than 2 or 5, then the decimal expression repeats.

example: 9/25

25 = 5*5, so it will be terminating

example: 7/12

12 = 3*2*2, which contains a 3, so it will be repeating.

. Find two polynomial expressions whose quotient, when simplified, is 1/x . Use that division problem to determine whether polynomials are closed under division.

Answers

Answer:

The two polynomials are:

(x + 1) and (x² + x)

Step-by-step explanation:

A polynomial is simply an expression which consists of variables & coefficients involving only the operations of addition, subtraction, multiplication, and non - negative integer exponents of variables.

Now, 1 and x are both polynomials. Thus; 1/x is already a quotient of a polynomial.

Now, to get two polynomial expressions whose quotient, when simplified, is 1/x, we will just multiply the numerator and denominator by the same polynomial to get more quotients.

So,

Let's multiply both numerator and denominator by (x + 1) to get;

(x + 1)/(x(x + 1))

This gives; (x + 1)/(x² + x)

Now, 1 and x are both polynomials but the expression "1/x" is not a polynomial but a quotient and thus polynomials are not closed under division.

A truck costs $8,000 with a residual value of $1,000. It has an estimated useful life of 7 years. If the truck was bought on July 9 what would be the book value at the end of year 1?

Answers

Answer:

$7520.55

Step-by-step explanation:

Cost of truck = $8000

Residual value = $1000

Estimated useful life = 7 years

Depreciation = (cost of asset - salvage value) / useful life

Depreciation = (8000 - 1000) / 7

Depreciation = 7000 / 7

Depreciation = $1000

Truck was purchased on July 9, Therefore, Depreciation by the end of year one will be;

Number of days between July 9 and year end = 175 days

Daily Depreciation = $1000 / 365 = $2.739

Total Depreciation by year end = (daily Depreciation * 175 days) = $479.45.

Book value at the end of year 1 = (8000 - 479.45)

= $7520.55

Answer:

The answer is "$7,500".

Step-by-step explanation:

Formula:

In the month of July-December the depreciation:

[tex]\frac{\text{Cost-Residual}}{\text{Useful life}}\times \frac{6}{12} \\[/tex]

Cost-Residual= costs -residual value

Given:

Cost-Residual = $8, 000 - $ 1,00

                        = $7,000

Useful life= 7 years

Put the value in the above-given formula:

[tex]=\frac{7 000}{7}\times \frac{6}{12}\\\\= 1 000\times \frac{1}{2}\\\\= 5 00\\[/tex]

Therefore, the book value on point at the end of one year is:

= $8,000 -$ 500

= $7,500

Can you please Solve for x

Answers

x - 3 = 27

add three to both sides

then x= 30

●✴︎✴︎✴︎✴︎✴︎✴︎✴︎✴︎❀✴︎✴︎✴︎✴︎✴︎✴︎✴︎✴︎✴︎●

         Hi my lil bunny!

❧⎯⎯⎯⎯⎯⎯⎯⎯⎯⎯⎯⎯⎯⎯⎯⎯⎯⎯⎯⎯⎯⎯⎯⎯⎯⎯⎯⎯⎯⎯⎯⎯⎯⎯⎯⎯⎯⎯☙

Let's solve your equation step-by-step.

[tex]x-3=27[/tex]

Step 1: Add 3 to both sides.

[tex]x -3 + 3 = 27 +3[/tex]

[tex]x = 30[/tex]

So the answer is : [tex]x = 30[/tex]

❧⎯⎯⎯⎯⎯⎯⎯⎯⎯⎯⎯⎯⎯⎯⎯⎯⎯⎯⎯⎯⎯⎯⎯⎯⎯⎯⎯⎯⎯⎯⎯⎯⎯⎯⎯⎯⎯⎯☙

●✴︎✴︎✴︎✴︎✴︎✴︎✴︎✴︎❀✴︎✴︎✴︎✴︎✴︎✴︎✴︎✴︎✴︎●

Hope this helped you.

Could you maybe give brainliest..?

❀*May*❀

Complete the following two-way frequency table.

Answers

Answer:

Step-by-step explanation:

Number of candies with Forest = 12

Candies containing coconut and chocolate both = Number common in coconut and the chocolate = 3

Candies which do not contain coconut but contain the chocolate = 6

Candies which contain the coconut but do not contain the chocolate = 1

Candies which neither contain the chocolate nor coconut = 2

From the given Venn diagram,

                                                Contain coconut         Do not contain coconut

Contain chocolate                              3                                       6

Do not contain chocolate                 1                                        2

What is 12.5% of 72

Answers

Answer:

[tex]\boxed{9}[/tex]

Step-by-step explanation:

[tex]\sf of \ refers \ to \ multiplication.[/tex]

[tex]12.5\% \times 72[/tex]

[tex]\frac{12.5}{100} \times 72[/tex]

[tex]\sf Multiply.[/tex]

[tex]\frac{900}{100} =9[/tex]

Greyson completes a dive from a
cliff 75-feet above a river. It takes
him only 1.5 seconds to hit the
water and then another 0.5
second to descend 10 feet into the river

what’s the x axis and y axis?

Answers

Answer: y: height, x: time.

Step-by-step explanation:

The data we have is:

The initial position of Greyson is 75ft above the river.

He needs 1.5 seconds to hit the water, and other 0.5s tho reach the bottom of the river.

Then we have a relationship of height vs time.

The y axis will represent the heigth of Greyson, and the x-axis will represent the time, such that at the time x = 0 seconds, we have y = 75ft

I need help on both answers. They’re different from my other problems so I’m kinda confused

Answers

i think it would be b but
i’m not sure about the other one

LCM of x<sup>2</sup>+5x+6 and x<sup>2</sup>-x-6 is ………………………





Answers

Answer:

[tex] (x^2 - 9)(x + 2) [/tex]

Step-by-step explanation:

Given:

[tex] x^2 + 5x + 6 [/tex]

[tex] x^2 - x - 6 [/tex]

Required:

LCM of the polynomials

SOLUTION:

Step 1: Factorise each polynomial

[tex] x^2 + 5x + 6 [/tex]

[tex] x^2 + 3x + 2x + 6 [/tex]

[tex] (x^2 + 3x) + (2x + 6) [/tex]

[tex] x(x + 3) + 2(x + 3) [/tex]

[tex] (x + 2)(x + 3) [/tex]

[tex] x^2 - x - 6 [/tex]

[tex] x^2 - 3x +2x - 6 [/tex]

[tex] x(x - 3) + 2(x - 3) [/tex]

[tex] (x + 2)(x - 3) [/tex]

Step 2: find the product of each factor that is common in both polynomials.

We have the following,

[tex] x^2 + 5x + 6 = (x + 2)(x + 3) [/tex]

[tex] x^2 - x - 6 = (x + 2)(x - 3) [/tex]

The common factors would be: =>

[tex] (x + 2) [/tex] (this is common in both polynomials, so we would take just one of them as a factor.

[tex] (x + 3) [/tex] and,

[tex] (x - 3) [/tex]

Their product = [tex] (x - 3)(x + 3)(x +2) = (x^2 - 9)(x + 2) [/tex]

Basic math for 20 points + brainliest!

Answers

Answer:

Look at photo

Step-by-step explanation:

What is the rate of change and initial value for the linear relation that includes the points shown in the table?
ху
1 | 20
3 | 10
5 | 0
7 | -10
A. Initial value: 20, rate of change: 10
B. Initial value: 30, rate of change: 10
C. Initial value: 25, rate of change: -5
D. Initial value: 20, rate of change: -10

Answers

Answer:

C, at 0/25, 1/20, 2/15, 3/10,...

Answer:

C

Step-by-step explanation:

Can any kind soul help me please



Use the cosine rule​

Answers

Answer:

M = 45°

Step-by-step explanation:

a²=b²+c²-2bc Cos A

m² = n² + p² - 2np cos M

(5.1)² = (7.2)² + (5.3)² - 2(7.2)(5.3) cos M

26.01 = 51.84 + 28.09 - 76.32 cos M

-53.92 = -76.32 cos M

0.7065 = cos M

M = [tex]cos^{-1 }[/tex](0.7065)

M = 45°

A shop sells DVDs and CDs.

DVDs are sold at one price.
CDs are sold at a different price.

2 DVDs and 1 CD cost £35
2 DVDs and 2 CDs cost £45

Martin has £50 Does he have enough to buy 1 DVD and 3 CDs?

Answers

Answer:

Step-by-step explanation:

Lets Price of a DVD is fixed i.e. 15

and One CD price is 5 (Not fixed)

In First situation

2 DVDs and 1 CD cost = 35 as given

2 x 15 + 5 = 35

Lets one CD price is 7.5

In Second situation

2 x 15 + 2 x 7.5 = 45

Its mean CD price may be between 5 to 7.5

In asked scenario, Martin has 50

1 DVD and 3 CDs?

1 x 15 + 3 x 7.5 = 37.5

37.5 is lesser than 50

Hence Martin has enough to buy 1 DVD and 3 CDs.

Z= -3 - 8i Find the angle θtheta (in degrees) that z makes in the complex plane. Round your answer, if necessary, to the nearest tenth. Express θtheta between -180 180 degrees.

Answers

Answer:  -110.6 degrees approximately

The angle is negative to indicate a clockwise rotation.

======================================================

Explanation:

Z = -3 - 8i is in the form z = a+bi with a = -3 and b = -8

In the complex plane the point (a,b) represents the location of z = a+bi

Define three points with the locations

P = (a,b) = (-3,8)

Q = (0,0)

R = (10,0)

The angle PQR is the angle theta we're looking for. This is the angle formed between the positive x axis and the terminal point (a,b)

Use the arctan function to find theta

theta = arctan(b/a)

theta = arctan( (-8)/(-3) )

theta = 69.4439547804166

theta = 69.4 degrees approximately

Note how this theta value is in quadrant Q1, but (a,b) = (-3, -8) is in Q3

So we need to add 180 degrees to adjust this error.

69.4+180 = 249.4

and we're now in the proper quadrant. We would stop here if your teacher did not put the restriction that theta must be between -180 and 180.

However, this restriction is in place so we need to find the difference of 360 and 249.4 to get 360-249.4 = 110.6

-----------

The angle 249.4 degrees is coterminal to -110.6 degrees. They both point in the same direction.

angle 249.4 degrees is found by starting pointing directly east and rotating 249.4 degrees counterclockwise

angle -110.6 degrees is found by starting directly east and rotating 110.6 degrees clockwise.

Check out the diagram below. I used GeoGebra to make the diagram.

Find the missing probability. P(A)=1120,P(B|A)=1320,P(A∩B)=?

Answers

Answer: 143/400

Explanation:

Assuming you meant to say

P(A) = 11/20

P(B|A) = 13/20

then,

P(A∩B) = P(A)*P(B|A)

P(A∩B) = (11/20)*(13/20)

P(A∩B) = (11*13)/(20*20)

P(A∩B) = 143/400

THE ANSWER IS 143/400 !

Find the measure of b.

Answers

Answer:

b = 80°

Step-by-step explanation:

The inscribed angle measuring 100°, is supplementary to the angle opposite it in the inscribed quadrilateral.

Thus, the angle is = 80°

Therefore, b + 80° = 180° (angle on a straight line = 180°).

Thus, b = 180° - 80° = 100°.

The measure of b is 80°.

The cost of milk is modeled by a linear equation where four quarts (one gallon) costs $3.09 while two quarts
(half-gallon) costs $1.65. Write the linear equation that expresses the price in terms of quarts. How much would
an eight-quart container of milk cost?

Answers

Answer:

linear equation to express the price is:

y=0.72x+0.21

An eight quarts will cost :  $5.97

Step-by-step explanation:

linear equation represent y=mx+b

let x=quarts ( x=4, x=2)

y= price (3.09 and y=1.65 )

two points (4,3.09) and (2,1.65)

need to find the slope m:

y2-y1/x2-x1

(1.65-3.09)/(2-4) ⇒ m=0.72

y=0.72x+b  find b at point (2,1.65)

1.65=0.72(2) +b  ⇒ b=0.21

y=0.72x +0.21

check : point (4,3.09)

y=0.72(4) +0.21

y=3.09  ( correct)

An eight quarts will cost :

y=0.72(8)+0.21

y=5.97 dollars

Can someone please tell me how to solve this problem??!! I literally have to go back in math if I don’t pass this HELP!!

Answers

Answer:

          D.   270° < φ < 360°

Step-by-step explanation:

Imagine coordinate system

I quarter is where x>0 and y>0  {right top} and it is (0°,90°)

II quarter is where x<0 and y>0  {left top} and it is (90°,180°)

III quarter is where x<0 and y<0  {left bottom} and it is (180°,270°)

IV quarter is where x>0 and y<0  {right bottom} and it is (270°,360°)

Now, we have an angle wich vertex is point (0,0) and one of its sides is X-axis and the second lay at one of the quarters.

For the trig functons of an angle created by this second side always are true:

In first quarter all functions are >0

in second one only sine

in third one: tangent and cotangent

and in fourth one: cosine

{You can check this by selecting any point on the second side of angle and put it's coordinates to formulas of these functions:

[tex]\sin \phi=\dfrac y{\sqrt{x^2+y^2}}\,,\quad \cos \phi=\dfrac x{\sqrt{x^2+y^2}}\,,\quad \tan\phi=\dfrac yx\,,\quad \cot\phi=\dfrac xy[/tex]  }

So:

sinφ<0  ⇒ III or IV quarter

tanφ<0  ⇒ I or IV quarter

IV quarter  ⇒  φ ∈ (270°, 360°)

HELP ASAP PLEASE!!!!!!!!!!!!!!!!!

Answers

Answer:

A

C

D

Step-by-step explanation:

√54 or√9 *√6 or √27 *√4

are equal to the answer.

You can do that by doing the square of outer number which is 3 which equals to 9 when squared and multiplying that with the number inside the square root.

Type the correct answer in the box. Use numerals instead of words. If necessary, use / for the fraction bar. Stacy goes to the county fair with her friends. The total cost of ride tickets is given by the equation c = 3.5t, where c is the total cost of tickets and t is the number of tickets. If Stacy bought 15 tickets, she would spend $

Answers

Answer:

$52.2

Step-by-step explanation:

Given her total cost of ride tickets modeled by the equation c = 3.5t where c is the total cost of tickets and t is the number of tickets, If Stacy bought 15 tickets, to know the amount she would spend on 15 tickets, we will substitute t = 15 into the modeled equation as shown;

[tex]c = 3.5t\\when t = 15\\\\c = 3.5(15)\\\\c = \frac{35}{10} * 15\\ \\c = \frac{5*7}{5*2} * 15\\\\[/tex]

[tex]c = \frac{7}{2} * 15\\ \\c = \frac{105}{2}\\ \\c = \ 52.2[/tex]

Hence Stacy would spend $52.2 on 15 tickets

Answer:

I hope this helps!

Step-by-step explanation:

Fill in the blank with a number to make the expression a perfect square.
u^2+8u+?

Answers

Answer:

16

Step-by-step explanation:

Hello, do you remember that result?

For any a and b real numbers,

[tex](a+b)^2=a+2\cdot a \cdot b+b^2[/tex]

In this example, we have.

[tex]u^2+8u=u^2+2\cdot 4 \cdot u\\\\\text{ This is the beginning of ... } u^2+8u+4^2=u^2+8u+16\\\\\text{ So, we need to add 16 to make a perfect square}\\\\u^2+8u+\boxed{16}=u^2+2\cdot 4\cdot u +4^2=(u+4)^2[/tex]

Hope this helps.

Do not hesitate if you need further explanation.

Thank you

Find the measure of each side indicated. Round to the nearest tenth.

A) 19.8
C) 24.9
B) 27.2
D) 25.3

Answers

Answer:

D. 25.3

Step-by-step explanation:

tan∅ = opposite over adjacent

Step 1: Write equation

tan66.5° = x/11

Step 2: Multiply both sides by 11

11tan66.5° = x

Step 3: Evaluate

x = 25.2983

x ≈ 25.3

Answer:

[tex]\huge\boxed{x = 25.3}[/tex]

Step-by-step explanation:

Tan θ = opposite / adjacent

Where θ = 66.5 , opposite = x and adjacent = 11

Tan 66.5 = x / 11

2.3 * 11 = x

25.3 = x

OR

x = 25.3

1: The best statement for reason 6 of this proof is -∠A ≅ ∠C
-∠B ≅ ∠D
-∠B and ∠D are supplements
-∠B ≅ ∠B

2.The best reason for statements 3.5. and 7 in this proof is
- Alternate interior angles are congruent.
-Corresponding angles are congruent.
-Alternate exterior angles are congruent.
-Interior angles on the same sides of a transversal are supplements.

3. The best statement for reason 8 of this proof is
-∠B ≅ ∠B -∠A and ∠C are supplements.
-∠B ≅ ∠D
-∠A ≅ ∠C

Answers

Answer:

1) -∠B ≅ ∠D

2) -Interior angles on the same side of a transversal are supplementary

3) -∠A ≅ ∠C

Step-by-step explanation:

1) Given that ∠A and ∠B are supplements and ∠A and ∠D are supplements, we have; ∠B ≅ ∠D

2)  Given that ABCD is a parallelogram, therefore ∠A and ∠B,  ∠A and ∠D and ∠B and ∠C  are interior angles on the same side of a transversal and are therefore supplementary

3) Given that ∠A and ∠B and ∠B and ∠C are supplementary, therefore, ∠A ≅ ∠C.


31. Each day, Talisa exercises by first
stretching and then swimming
some laps, as shown in the table.
Make a scatter plot of the total
time she exercises as a function
of the number of laps she swims.
Draw a trend line.

Answers

Answer:

Step-by-step explanation:

Given the following :

Laps - - - - - - - - 5 - - - 6 - - - 7 - - - 8 - - - 9

Total time - - - 25 - - 28 - - 29 - - 30 - - 32

Using online graphing tool:

The y - axis named dependent variable represents the total time taken.

The x-axis, represents the number of laps.

The equation of the trend line attached to the plot is in the form :

y = mx + c

y = 1.6x + 17.6

Where y = total time taken

x = number of laps

m = 1.6 = gradient of the line (change in y / change in x)

C = 17.6 = intercept (whee the trndline intersects the y-axis).

At the Olympic games, many events have several rounds of competition. One of these events is the men's 100 100100-meter backstroke. The upper dot plot shows the times (in seconds) of the top 8 88 finishers in the final round of the 2012 20122012 Olympics. The lower dot plot shows the times of the same 8 88 swimmers, but in the semifinal round. Which pieces of information can be gathered from these dot plots? (Remember that lower swim times are faster.) Choose all answers that apply: Choose all answers that apply:

Answers

Answer:

The center of the semifinal round distribution is greater than the center of final round distribution.

The variability in the semifinal round distribution is less than variability in the final round distribution.

Step-by-step explanation:

The mean value of each distribution set is not calculates as the center of semifinal round distribution is greater than the final round distribution. MAD Mean Absolute Deviation is calculated from the dotted graph plot, the distribution of semifinal round is less spread out than the final round distribution.

Answer:

correct answer is None of the above i understood nothing the other person was trying to say...

Step-by-step explanation:

mark me brainliest please...

Please solve (will make brainiest)

Answers

Answer:

1a) 1/64

1b) 1/169

1c) 1/9

Step-by-step explanation:

You have to apply Indices Law :

[tex] {a}^{ - n} = \frac{1}{ {a}^{n} } [/tex]

Question A,

[tex] {4}^{ - 3} = \frac{1}{ {4}^{3} } = \frac{1}{64} [/tex]

Question B,

[tex] {13}^{ - 2} = \frac{1}{ {13}^{2} } = \frac{1}{169} [/tex]

Question C,

[tex] {( - 3)}^{ - 2} = {( - \frac{1}{3}) }^{2} = \frac{1}{9} [/tex]

A laundry basket contains 18 blue socks and 24 black socks. What is the probability of randomly picking 2 black socks, with replacement, from the basket?

Answers

Answer:

144/441

Step-by-step explanation:

There are 18+24=42 total socks

There are 24 black socks

So the probability is (24/42)*(24/42)=12/21 * 12/21 = 144/441

Answer:

189

Step-by-step explanation:

Other Questions
(Please help will give brainliest)Which scenario goes with the vocabulary word cubrir?Mara tiene picor de la piel.Gustavo estornuda mucho.Laura necesita ir al terapeuta.Felipe bebe agua para estar saludable. At a constant temperature, a sample of a gas in a balloon that originally had a volume of 5.00 L and pressure of 626 torr has its volume changed to 6.72 L. Calculate the new pressure in torr. True or false? induction is a kind of thinking you use to form general ideas and rules based on mathematical formuals 1/9, -0.1, -2/12 in order Psychosocial development in late adulthood is best understood by looking at clusters of theories such as:____________. During the previous weeks, you have developed your public speaking skills. As you evaluate your progress at midterm, which one of the skills listed below do you plan to further develop during the remainder of the quarter? Thoroughly explain why you selected this skill and share specific actions you will take to further develop it. Actively listening Conducting research Creating an outline ICOT Industries issued 28 million of its $1 par common shares for $492 million on April 11. Legal, promotional, and accounting services necessary to effect the sale cost $3 million. Required: 1. Prepare the journal entry to record the issuance of the shares. (If no entry is required for a transaction/event, select "No journal entry required" in the first account field. Enter your answers in millions (i.e., 10,000,000 should be entered as 10).) Use the discriminant to determine the number of real solutions to the equation. 8m^2+2m=0 A charge is distributed uniformly along a long straight wire. The electric field 2 cm from the wire is 36 N/C. The electric field 4 cm from the wire is: Political uncertainty is the risk associated with: a. policies affecting foreign companies. b. social unrest. c. changes in government policies. d. changes in law. A museum curator is hanging 7 paintings in a row for an exhibit. There are 4 Renaissance paintings and 3 Baroque paintings. From left to right, all of the Renaissance paintings will be hung first, followed by all of the Baroque paintings. How many ways are there to hang the paintings 1. What historical event is the team investigating? What, if anything, do you alreadyknow about this event (The wreck of the Sultana steamboat)? What the relation of 1/c=1/c1+1/c2 hence find c Select the correct answer. Rita is a registered dietician. What does her work entail? A. prescribing medication for clients B. cooking healthy meals for students C. demonstrating how to use gym equipment D. making recommendations for healthy eating habits The mean weight of newborn infants at a community hospital is 6.6 pounds. A sample of seven infants is randomly selected and their weights at birth are recorded as 9.0, 7.3, 6.0, 8.8, 6.8, 8.4, and 6.6 pounds. Does the sample data show a significant increase in the average birthrate at a 5% level of significance? A. Fail to reject the null hypothesis and conclude the mean is 6.6 lb. B. Reject the null hypothesis and conclude the mean is lower than 6.6 lb. C. Reject the null hypothesis and conclude the mean is greater than 6.6 lb. D. Cannot calculate because the population standard deviation is unknown Your mother has left you in charge of the annual family yard sale. Before she leaves you to your entrepreneurial abilities, she explains that she has made the job easy for you: everything costs either $1.50 or $3.50. She asks you to keep track of how many of each type of item is sold, and you make a list, but it gets lost sometime throughout the day. Just before shes supposed to get home, you realize that all you know is that there were 150 items to start with (your mom counted) and you have 41 items left. Also, you know that you made $227.50. Write a system of equations that you could solve to figure out how many of each type of item you sold.A) x + y = 109(1.5)x + 227.50 = (3.5)yB) x + y = 109(3.5)x + 227.50 = (1.5)yC) x + y = 41(1.5)x + 227.50 = (3.5)yD) x + y = 109(1.5)x + (3.5)y = 227.50E) x + y = 150(1.5)x + (3.5)y = 227.50F) x + y = $3.50(1.5)x + (3.5)y = 227.50 Cameroon Corp. manufactures and sells electric staplers for $15.30 each. If 10,000 units were sold in December, and management forecasts 3.3% growth in sales each month, the number of electric stapler sales budgeted for March should be: A chemist fills a reaction vessel with 0.978 g aluminum hydroxide AlOH3 solid, 0.607 M aluminum Al+3 aqueous solution, and 0.396 M hydroxide OH aqueous solution at a temperature of 25.0C. Under these conditions, calculate the reaction free energy G for the following chemical reaction:Al(OH)3(s) = A1+ (aq) +30H (aq) Use the thermodynamic information in the ALEKS Data tab. Round your answer to the nearest kilojoule.KJ Item9 2 points Time Remaining 2 hours 55 minutes 49 seconds02:55:49 eBookItem 9Item 9 2 points Time Remaining 2 hours 55 minutes 49 seconds02:55:49 TB MC Qu. 6-143 Keyser Corporation, which has... Keyser Corporation, which has only one product, has provided the following data concerning its most recent month of operations: Selling price$118 Units in beginning inventory 400 Units produced 2,100 Units sold 2,300 Units in ending inventory 200 Variable costs per unit: Direct materials$37 Direct labor$23 Variable manufacturing overhead$3 Variable selling and administrative expense$5 Fixed costs: Fixed manufacturing overhead$73,500 Fixed selling and administrative expense$29,900 The company produces the same number of units every month, although the sales in units vary from month to month. The company's variable costs per unit and total fixed costs have been constant from month to month. What is the net operating income for the month under variable costing? what does rise and fall of a nation rests with every one of its citizens meaning?